0 Daumen
357 Aufrufe

stehe vor folgender Aufgabe:

Sei für $$ n \in \mathbb{N_0} $$

$$ H_n(x) := (-1)^n e^{x^2} \frac{d^n}{dx^n} (e^{-x^2}) , x \in \mathbb{R} $$

das n-te Hermite Polynom. Zeigen Sie für $$ n,m \in \mathbb{N_0} $$ und $$ n \neq m  $$

$$ \int_{- \infty}^{\infty} H_n (x) H_m (x) e^{-x^2} dx = 0 $$


Problem/Ansatz:

Als Hinweis wurde die Verwendung der sukzessiven partiellen Integration gegeben, ich weiß zwar wie diese funktioniert, jedoch nicht wie ich sie hier ansengend soll, bzw. wie ich das überhaupt ableiten soll.


:)

von

Ein anderes Problem?

Stell deine Frage

Willkommen bei der Mathelounge! Stell deine Frage einfach und kostenlos

x
Made by a lovely community